Permutations and terminology












1












$begingroup$


Say I have the following permutation



$$sigma ={begin{pmatrix}1&2&3&4&5&6&7&8\1&2&3&8&4&5&6&7end{pmatrix}}$$



which consists to let unchanged some first elements from $1$ to $k$ and to apply a circular shift on the elements ${k+1, ldots, n}$.



What is the name for such transformation?



Thank you










share|cite|improve this question











$endgroup$








  • 2




    $begingroup$
    The name is "a $5$-cycle in $S_8$", namely $sigma=(48765)$.
    $endgroup$
    – Dietrich Burde
    Dec 2 '18 at 19:10












  • $begingroup$
    Thank you @DietrichBurde Does this (n-k)-cycle let unchanged the first $k$ elements?
    $endgroup$
    – Adam54
    Dec 2 '18 at 19:19












  • $begingroup$
    Yes, it keeps $1$,$2$, $3$ fixed (unchanged). Have a look at the cycle notation for permutations.
    $endgroup$
    – Dietrich Burde
    Dec 2 '18 at 19:20








  • 1




    $begingroup$
    Have a look at the cycle notation: $(45)$ means that $4$ goes to $5$ and $5$ goes to $4$. Then $(48765)$ means that $4mapsto 8mapsto 7mapsto 6mapsto 5mapsto 4mapstocdots $
    $endgroup$
    – Dietrich Burde
    Dec 2 '18 at 19:23








  • 1




    $begingroup$
    You should read en.wikipedia.org/wiki/Cyclic_permutation and en.wikipedia.org/wiki/Permutation#Cycle_notation
    $endgroup$
    – Jean-Claude Arbaut
    Dec 2 '18 at 19:31
















1












$begingroup$


Say I have the following permutation



$$sigma ={begin{pmatrix}1&2&3&4&5&6&7&8\1&2&3&8&4&5&6&7end{pmatrix}}$$



which consists to let unchanged some first elements from $1$ to $k$ and to apply a circular shift on the elements ${k+1, ldots, n}$.



What is the name for such transformation?



Thank you










share|cite|improve this question











$endgroup$








  • 2




    $begingroup$
    The name is "a $5$-cycle in $S_8$", namely $sigma=(48765)$.
    $endgroup$
    – Dietrich Burde
    Dec 2 '18 at 19:10












  • $begingroup$
    Thank you @DietrichBurde Does this (n-k)-cycle let unchanged the first $k$ elements?
    $endgroup$
    – Adam54
    Dec 2 '18 at 19:19












  • $begingroup$
    Yes, it keeps $1$,$2$, $3$ fixed (unchanged). Have a look at the cycle notation for permutations.
    $endgroup$
    – Dietrich Burde
    Dec 2 '18 at 19:20








  • 1




    $begingroup$
    Have a look at the cycle notation: $(45)$ means that $4$ goes to $5$ and $5$ goes to $4$. Then $(48765)$ means that $4mapsto 8mapsto 7mapsto 6mapsto 5mapsto 4mapstocdots $
    $endgroup$
    – Dietrich Burde
    Dec 2 '18 at 19:23








  • 1




    $begingroup$
    You should read en.wikipedia.org/wiki/Cyclic_permutation and en.wikipedia.org/wiki/Permutation#Cycle_notation
    $endgroup$
    – Jean-Claude Arbaut
    Dec 2 '18 at 19:31














1












1








1





$begingroup$


Say I have the following permutation



$$sigma ={begin{pmatrix}1&2&3&4&5&6&7&8\1&2&3&8&4&5&6&7end{pmatrix}}$$



which consists to let unchanged some first elements from $1$ to $k$ and to apply a circular shift on the elements ${k+1, ldots, n}$.



What is the name for such transformation?



Thank you










share|cite|improve this question











$endgroup$




Say I have the following permutation



$$sigma ={begin{pmatrix}1&2&3&4&5&6&7&8\1&2&3&8&4&5&6&7end{pmatrix}}$$



which consists to let unchanged some first elements from $1$ to $k$ and to apply a circular shift on the elements ${k+1, ldots, n}$.



What is the name for such transformation?



Thank you







permutations permutation-cycles






share|cite|improve this question















share|cite|improve this question













share|cite|improve this question




share|cite|improve this question








edited Dec 2 '18 at 19:25









Bernard

119k740113




119k740113










asked Dec 2 '18 at 19:09









Adam54Adam54

515




515








  • 2




    $begingroup$
    The name is "a $5$-cycle in $S_8$", namely $sigma=(48765)$.
    $endgroup$
    – Dietrich Burde
    Dec 2 '18 at 19:10












  • $begingroup$
    Thank you @DietrichBurde Does this (n-k)-cycle let unchanged the first $k$ elements?
    $endgroup$
    – Adam54
    Dec 2 '18 at 19:19












  • $begingroup$
    Yes, it keeps $1$,$2$, $3$ fixed (unchanged). Have a look at the cycle notation for permutations.
    $endgroup$
    – Dietrich Burde
    Dec 2 '18 at 19:20








  • 1




    $begingroup$
    Have a look at the cycle notation: $(45)$ means that $4$ goes to $5$ and $5$ goes to $4$. Then $(48765)$ means that $4mapsto 8mapsto 7mapsto 6mapsto 5mapsto 4mapstocdots $
    $endgroup$
    – Dietrich Burde
    Dec 2 '18 at 19:23








  • 1




    $begingroup$
    You should read en.wikipedia.org/wiki/Cyclic_permutation and en.wikipedia.org/wiki/Permutation#Cycle_notation
    $endgroup$
    – Jean-Claude Arbaut
    Dec 2 '18 at 19:31














  • 2




    $begingroup$
    The name is "a $5$-cycle in $S_8$", namely $sigma=(48765)$.
    $endgroup$
    – Dietrich Burde
    Dec 2 '18 at 19:10












  • $begingroup$
    Thank you @DietrichBurde Does this (n-k)-cycle let unchanged the first $k$ elements?
    $endgroup$
    – Adam54
    Dec 2 '18 at 19:19












  • $begingroup$
    Yes, it keeps $1$,$2$, $3$ fixed (unchanged). Have a look at the cycle notation for permutations.
    $endgroup$
    – Dietrich Burde
    Dec 2 '18 at 19:20








  • 1




    $begingroup$
    Have a look at the cycle notation: $(45)$ means that $4$ goes to $5$ and $5$ goes to $4$. Then $(48765)$ means that $4mapsto 8mapsto 7mapsto 6mapsto 5mapsto 4mapstocdots $
    $endgroup$
    – Dietrich Burde
    Dec 2 '18 at 19:23








  • 1




    $begingroup$
    You should read en.wikipedia.org/wiki/Cyclic_permutation and en.wikipedia.org/wiki/Permutation#Cycle_notation
    $endgroup$
    – Jean-Claude Arbaut
    Dec 2 '18 at 19:31








2




2




$begingroup$
The name is "a $5$-cycle in $S_8$", namely $sigma=(48765)$.
$endgroup$
– Dietrich Burde
Dec 2 '18 at 19:10






$begingroup$
The name is "a $5$-cycle in $S_8$", namely $sigma=(48765)$.
$endgroup$
– Dietrich Burde
Dec 2 '18 at 19:10














$begingroup$
Thank you @DietrichBurde Does this (n-k)-cycle let unchanged the first $k$ elements?
$endgroup$
– Adam54
Dec 2 '18 at 19:19






$begingroup$
Thank you @DietrichBurde Does this (n-k)-cycle let unchanged the first $k$ elements?
$endgroup$
– Adam54
Dec 2 '18 at 19:19














$begingroup$
Yes, it keeps $1$,$2$, $3$ fixed (unchanged). Have a look at the cycle notation for permutations.
$endgroup$
– Dietrich Burde
Dec 2 '18 at 19:20






$begingroup$
Yes, it keeps $1$,$2$, $3$ fixed (unchanged). Have a look at the cycle notation for permutations.
$endgroup$
– Dietrich Burde
Dec 2 '18 at 19:20






1




1




$begingroup$
Have a look at the cycle notation: $(45)$ means that $4$ goes to $5$ and $5$ goes to $4$. Then $(48765)$ means that $4mapsto 8mapsto 7mapsto 6mapsto 5mapsto 4mapstocdots $
$endgroup$
– Dietrich Burde
Dec 2 '18 at 19:23






$begingroup$
Have a look at the cycle notation: $(45)$ means that $4$ goes to $5$ and $5$ goes to $4$. Then $(48765)$ means that $4mapsto 8mapsto 7mapsto 6mapsto 5mapsto 4mapstocdots $
$endgroup$
– Dietrich Burde
Dec 2 '18 at 19:23






1




1




$begingroup$
You should read en.wikipedia.org/wiki/Cyclic_permutation and en.wikipedia.org/wiki/Permutation#Cycle_notation
$endgroup$
– Jean-Claude Arbaut
Dec 2 '18 at 19:31




$begingroup$
You should read en.wikipedia.org/wiki/Cyclic_permutation and en.wikipedia.org/wiki/Permutation#Cycle_notation
$endgroup$
– Jean-Claude Arbaut
Dec 2 '18 at 19:31










1 Answer
1






active

oldest

votes


















0












$begingroup$

$1, 2, 3$ each map to themselves. $1to 1$. Period. $2to 2.$ Period. $3to 3.$ Period. We can represent this as $(1)(2)(3)$, but typically this is not customary.



So we can start with $4$:



$4 to 8$, then



$8 to 7$, then



$7to 6$,



then $6to 5$, and finally,



$5 to 4$, completing the five cycle (bringing us back to the starting number for the cycle, $4$:



That gives us the permutation $sigma = (48765)$. This can also be written as $$sigma = (1)(2)(3)(48765) = (48765)$$





To get the cycle you are looking for, you'll need the following permutation:



$alpha ={begin{pmatrix}1&2&3&4&5&6&7&8\1&2&3&5&6&7&8&4end{pmatrix}} = (45678)$



The name for this transformation is merely "permutation" $alpha$ on the set $S$ of $8$ numbers, specifically, both $alpha, sigma in mathbb S_8$, the group of permutations on $S$.



If we were talking about vertices of an octagon, numbered 1-8, then $(12345678)$ could be described as a rotation of the octagon.






share|cite|improve this answer











$endgroup$













  • $begingroup$
    Thanks @amWhy As I have written to Dietrich, I think I am reffering to another kind of cycle.
    $endgroup$
    – Adam54
    Dec 2 '18 at 19:29










  • $begingroup$
    Top number goes to bottom number in each column.
    $endgroup$
    – amWhy
    Dec 2 '18 at 19:37










  • $begingroup$
    Thank you. So writing left shift is straightforward, but for writing a right shift we need to read from top to bottom?
    $endgroup$
    – Adam54
    Dec 2 '18 at 19:42










  • $begingroup$
    Yes, the numbers in the top row represent the 8 possible numbers to permute. Where each of those numbers goes is written immediately below the number, in the second row.
    $endgroup$
    – amWhy
    Dec 2 '18 at 19:46










  • $begingroup$
    Thanks again. So, if I have a sequence of 8 unknown numbers, but I know that when I will have these numbers, I will right shift the 5 last numbers. How can I write this function, terminologically?
    $endgroup$
    – Adam54
    Dec 2 '18 at 19:49











Your Answer





StackExchange.ifUsing("editor", function () {
return StackExchange.using("mathjaxEditing", function () {
StackExchange.MarkdownEditor.creationCallbacks.add(function (editor, postfix) {
StackExchange.mathjaxEditing.prepareWmdForMathJax(editor, postfix, [["$", "$"], ["\\(","\\)"]]);
});
});
}, "mathjax-editing");

StackExchange.ready(function() {
var channelOptions = {
tags: "".split(" "),
id: "69"
};
initTagRenderer("".split(" "), "".split(" "), channelOptions);

StackExchange.using("externalEditor", function() {
// Have to fire editor after snippets, if snippets enabled
if (StackExchange.settings.snippets.snippetsEnabled) {
StackExchange.using("snippets", function() {
createEditor();
});
}
else {
createEditor();
}
});

function createEditor() {
StackExchange.prepareEditor({
heartbeatType: 'answer',
autoActivateHeartbeat: false,
convertImagesToLinks: true,
noModals: true,
showLowRepImageUploadWarning: true,
reputationToPostImages: 10,
bindNavPrevention: true,
postfix: "",
imageUploader: {
brandingHtml: "Powered by u003ca class="icon-imgur-white" href="https://imgur.com/"u003eu003c/au003e",
contentPolicyHtml: "User contributions licensed under u003ca href="https://creativecommons.org/licenses/by-sa/3.0/"u003ecc by-sa 3.0 with attribution requiredu003c/au003e u003ca href="https://stackoverflow.com/legal/content-policy"u003e(content policy)u003c/au003e",
allowUrls: true
},
noCode: true, onDemand: true,
discardSelector: ".discard-answer"
,immediatelyShowMarkdownHelp:true
});


}
});














draft saved

draft discarded


















StackExchange.ready(
function () {
StackExchange.openid.initPostLogin('.new-post-login', 'https%3a%2f%2fmath.stackexchange.com%2fquestions%2f3023056%2fpermutations-and-terminology%23new-answer', 'question_page');
}
);

Post as a guest















Required, but never shown

























1 Answer
1






active

oldest

votes








1 Answer
1






active

oldest

votes









active

oldest

votes






active

oldest

votes









0












$begingroup$

$1, 2, 3$ each map to themselves. $1to 1$. Period. $2to 2.$ Period. $3to 3.$ Period. We can represent this as $(1)(2)(3)$, but typically this is not customary.



So we can start with $4$:



$4 to 8$, then



$8 to 7$, then



$7to 6$,



then $6to 5$, and finally,



$5 to 4$, completing the five cycle (bringing us back to the starting number for the cycle, $4$:



That gives us the permutation $sigma = (48765)$. This can also be written as $$sigma = (1)(2)(3)(48765) = (48765)$$





To get the cycle you are looking for, you'll need the following permutation:



$alpha ={begin{pmatrix}1&2&3&4&5&6&7&8\1&2&3&5&6&7&8&4end{pmatrix}} = (45678)$



The name for this transformation is merely "permutation" $alpha$ on the set $S$ of $8$ numbers, specifically, both $alpha, sigma in mathbb S_8$, the group of permutations on $S$.



If we were talking about vertices of an octagon, numbered 1-8, then $(12345678)$ could be described as a rotation of the octagon.






share|cite|improve this answer











$endgroup$













  • $begingroup$
    Thanks @amWhy As I have written to Dietrich, I think I am reffering to another kind of cycle.
    $endgroup$
    – Adam54
    Dec 2 '18 at 19:29










  • $begingroup$
    Top number goes to bottom number in each column.
    $endgroup$
    – amWhy
    Dec 2 '18 at 19:37










  • $begingroup$
    Thank you. So writing left shift is straightforward, but for writing a right shift we need to read from top to bottom?
    $endgroup$
    – Adam54
    Dec 2 '18 at 19:42










  • $begingroup$
    Yes, the numbers in the top row represent the 8 possible numbers to permute. Where each of those numbers goes is written immediately below the number, in the second row.
    $endgroup$
    – amWhy
    Dec 2 '18 at 19:46










  • $begingroup$
    Thanks again. So, if I have a sequence of 8 unknown numbers, but I know that when I will have these numbers, I will right shift the 5 last numbers. How can I write this function, terminologically?
    $endgroup$
    – Adam54
    Dec 2 '18 at 19:49
















0












$begingroup$

$1, 2, 3$ each map to themselves. $1to 1$. Period. $2to 2.$ Period. $3to 3.$ Period. We can represent this as $(1)(2)(3)$, but typically this is not customary.



So we can start with $4$:



$4 to 8$, then



$8 to 7$, then



$7to 6$,



then $6to 5$, and finally,



$5 to 4$, completing the five cycle (bringing us back to the starting number for the cycle, $4$:



That gives us the permutation $sigma = (48765)$. This can also be written as $$sigma = (1)(2)(3)(48765) = (48765)$$





To get the cycle you are looking for, you'll need the following permutation:



$alpha ={begin{pmatrix}1&2&3&4&5&6&7&8\1&2&3&5&6&7&8&4end{pmatrix}} = (45678)$



The name for this transformation is merely "permutation" $alpha$ on the set $S$ of $8$ numbers, specifically, both $alpha, sigma in mathbb S_8$, the group of permutations on $S$.



If we were talking about vertices of an octagon, numbered 1-8, then $(12345678)$ could be described as a rotation of the octagon.






share|cite|improve this answer











$endgroup$













  • $begingroup$
    Thanks @amWhy As I have written to Dietrich, I think I am reffering to another kind of cycle.
    $endgroup$
    – Adam54
    Dec 2 '18 at 19:29










  • $begingroup$
    Top number goes to bottom number in each column.
    $endgroup$
    – amWhy
    Dec 2 '18 at 19:37










  • $begingroup$
    Thank you. So writing left shift is straightforward, but for writing a right shift we need to read from top to bottom?
    $endgroup$
    – Adam54
    Dec 2 '18 at 19:42










  • $begingroup$
    Yes, the numbers in the top row represent the 8 possible numbers to permute. Where each of those numbers goes is written immediately below the number, in the second row.
    $endgroup$
    – amWhy
    Dec 2 '18 at 19:46










  • $begingroup$
    Thanks again. So, if I have a sequence of 8 unknown numbers, but I know that when I will have these numbers, I will right shift the 5 last numbers. How can I write this function, terminologically?
    $endgroup$
    – Adam54
    Dec 2 '18 at 19:49














0












0








0





$begingroup$

$1, 2, 3$ each map to themselves. $1to 1$. Period. $2to 2.$ Period. $3to 3.$ Period. We can represent this as $(1)(2)(3)$, but typically this is not customary.



So we can start with $4$:



$4 to 8$, then



$8 to 7$, then



$7to 6$,



then $6to 5$, and finally,



$5 to 4$, completing the five cycle (bringing us back to the starting number for the cycle, $4$:



That gives us the permutation $sigma = (48765)$. This can also be written as $$sigma = (1)(2)(3)(48765) = (48765)$$





To get the cycle you are looking for, you'll need the following permutation:



$alpha ={begin{pmatrix}1&2&3&4&5&6&7&8\1&2&3&5&6&7&8&4end{pmatrix}} = (45678)$



The name for this transformation is merely "permutation" $alpha$ on the set $S$ of $8$ numbers, specifically, both $alpha, sigma in mathbb S_8$, the group of permutations on $S$.



If we were talking about vertices of an octagon, numbered 1-8, then $(12345678)$ could be described as a rotation of the octagon.






share|cite|improve this answer











$endgroup$



$1, 2, 3$ each map to themselves. $1to 1$. Period. $2to 2.$ Period. $3to 3.$ Period. We can represent this as $(1)(2)(3)$, but typically this is not customary.



So we can start with $4$:



$4 to 8$, then



$8 to 7$, then



$7to 6$,



then $6to 5$, and finally,



$5 to 4$, completing the five cycle (bringing us back to the starting number for the cycle, $4$:



That gives us the permutation $sigma = (48765)$. This can also be written as $$sigma = (1)(2)(3)(48765) = (48765)$$





To get the cycle you are looking for, you'll need the following permutation:



$alpha ={begin{pmatrix}1&2&3&4&5&6&7&8\1&2&3&5&6&7&8&4end{pmatrix}} = (45678)$



The name for this transformation is merely "permutation" $alpha$ on the set $S$ of $8$ numbers, specifically, both $alpha, sigma in mathbb S_8$, the group of permutations on $S$.



If we were talking about vertices of an octagon, numbered 1-8, then $(12345678)$ could be described as a rotation of the octagon.







share|cite|improve this answer














share|cite|improve this answer



share|cite|improve this answer








edited Dec 31 '18 at 21:32

























answered Dec 2 '18 at 19:26









amWhyamWhy

1




1












  • $begingroup$
    Thanks @amWhy As I have written to Dietrich, I think I am reffering to another kind of cycle.
    $endgroup$
    – Adam54
    Dec 2 '18 at 19:29










  • $begingroup$
    Top number goes to bottom number in each column.
    $endgroup$
    – amWhy
    Dec 2 '18 at 19:37










  • $begingroup$
    Thank you. So writing left shift is straightforward, but for writing a right shift we need to read from top to bottom?
    $endgroup$
    – Adam54
    Dec 2 '18 at 19:42










  • $begingroup$
    Yes, the numbers in the top row represent the 8 possible numbers to permute. Where each of those numbers goes is written immediately below the number, in the second row.
    $endgroup$
    – amWhy
    Dec 2 '18 at 19:46










  • $begingroup$
    Thanks again. So, if I have a sequence of 8 unknown numbers, but I know that when I will have these numbers, I will right shift the 5 last numbers. How can I write this function, terminologically?
    $endgroup$
    – Adam54
    Dec 2 '18 at 19:49


















  • $begingroup$
    Thanks @amWhy As I have written to Dietrich, I think I am reffering to another kind of cycle.
    $endgroup$
    – Adam54
    Dec 2 '18 at 19:29










  • $begingroup$
    Top number goes to bottom number in each column.
    $endgroup$
    – amWhy
    Dec 2 '18 at 19:37










  • $begingroup$
    Thank you. So writing left shift is straightforward, but for writing a right shift we need to read from top to bottom?
    $endgroup$
    – Adam54
    Dec 2 '18 at 19:42










  • $begingroup$
    Yes, the numbers in the top row represent the 8 possible numbers to permute. Where each of those numbers goes is written immediately below the number, in the second row.
    $endgroup$
    – amWhy
    Dec 2 '18 at 19:46










  • $begingroup$
    Thanks again. So, if I have a sequence of 8 unknown numbers, but I know that when I will have these numbers, I will right shift the 5 last numbers. How can I write this function, terminologically?
    $endgroup$
    – Adam54
    Dec 2 '18 at 19:49
















$begingroup$
Thanks @amWhy As I have written to Dietrich, I think I am reffering to another kind of cycle.
$endgroup$
– Adam54
Dec 2 '18 at 19:29




$begingroup$
Thanks @amWhy As I have written to Dietrich, I think I am reffering to another kind of cycle.
$endgroup$
– Adam54
Dec 2 '18 at 19:29












$begingroup$
Top number goes to bottom number in each column.
$endgroup$
– amWhy
Dec 2 '18 at 19:37




$begingroup$
Top number goes to bottom number in each column.
$endgroup$
– amWhy
Dec 2 '18 at 19:37












$begingroup$
Thank you. So writing left shift is straightforward, but for writing a right shift we need to read from top to bottom?
$endgroup$
– Adam54
Dec 2 '18 at 19:42




$begingroup$
Thank you. So writing left shift is straightforward, but for writing a right shift we need to read from top to bottom?
$endgroup$
– Adam54
Dec 2 '18 at 19:42












$begingroup$
Yes, the numbers in the top row represent the 8 possible numbers to permute. Where each of those numbers goes is written immediately below the number, in the second row.
$endgroup$
– amWhy
Dec 2 '18 at 19:46




$begingroup$
Yes, the numbers in the top row represent the 8 possible numbers to permute. Where each of those numbers goes is written immediately below the number, in the second row.
$endgroup$
– amWhy
Dec 2 '18 at 19:46












$begingroup$
Thanks again. So, if I have a sequence of 8 unknown numbers, but I know that when I will have these numbers, I will right shift the 5 last numbers. How can I write this function, terminologically?
$endgroup$
– Adam54
Dec 2 '18 at 19:49




$begingroup$
Thanks again. So, if I have a sequence of 8 unknown numbers, but I know that when I will have these numbers, I will right shift the 5 last numbers. How can I write this function, terminologically?
$endgroup$
– Adam54
Dec 2 '18 at 19:49


















draft saved

draft discarded




















































Thanks for contributing an answer to Mathematics Stack Exchange!


  • Please be sure to answer the question. Provide details and share your research!

But avoid



  • Asking for help, clarification, or responding to other answers.

  • Making statements based on opinion; back them up with references or personal experience.


Use MathJax to format equations. MathJax reference.


To learn more, see our tips on writing great answers.




draft saved


draft discarded














StackExchange.ready(
function () {
StackExchange.openid.initPostLogin('.new-post-login', 'https%3a%2f%2fmath.stackexchange.com%2fquestions%2f3023056%2fpermutations-and-terminology%23new-answer', 'question_page');
}
);

Post as a guest















Required, but never shown





















































Required, but never shown














Required, but never shown












Required, but never shown







Required, but never shown

































Required, but never shown














Required, but never shown












Required, but never shown







Required, but never shown







Popular posts from this blog

Le Mesnil-Réaume

Ida-Boy-Ed-Garten

web3.py web3.isConnected() returns false always